PT1.S3.Q7 - A population of game ducks at a western lake...

diljotgill.rsdiljotgill.rs Member
edited December 2021 in Logical Reasoning 13 karma

Hi,

Could someone kindly help me understand why/how A is the answer for this question if my interpretation below is incorrect?

I got this wrong. I picked C, but now I think I see why A is right and why C is is wrong. Is C wrong because it is merely stating a fact stated by the stimulus - that there will be more males to x # of females at both lakes therefore it's not some we are 'inferring'?

And, is A right because given the sex ratio information re: females and males in the stimulus, we know there will always be a lower percentage of adult males at Western than at Eastern Lake?

Any help is appreciated!

Admin Note: Edited titled. Please use the format "PT#.S#.Q# - brief description of question"

Comments

  • Ashley2018-1Ashley2018-1 Live Member
    2249 karma

    There's only one question you need to ask and answer yourself to get this right: which lake has the greater disparity in the ratio between male and female ducks?

  • diljotgill.rsdiljotgill.rs Member
    13 karma

    Thank you so much!

Sign In or Register to comment.